LSAT and Law School Admissions Forum

Get expert LSAT preparation and law school admissions advice from PowerScore Test Preparation.

 jlam061695
  • Posts: 62
  • Joined: Sep 17, 2016
|
#31033
I can understand the reasoning behind D, but why is B incorrect? Doesn't B support the conclusion by stating that even if the cause (volume of collected recyclable stays the same) were gone, the effect would still happen?
User avatar
 Jonathan Evans
PowerScore Staff
  • PowerScore Staff
  • Posts: 726
  • Joined: Jun 09, 2016
|
#31082
Hi, JLam,

Good question. The important aspect of this question stem to note is: "most weakens"

Notice the kind of "hedge your bets" language in Answer Choice (B). Well maybe it will; maybe it won't. Who knows? It's possible that even if the volume goes up, it might not be cost effective.

Well, for one, that's a far cry from making the city's plan "absurd." It might illustrate one eventuality that could result in the plan not working, but remember, we're looking for what "most weakens."

Answer Choice (D) is a direct hit on the validity of this argument by showing a definite reason why a weekly schedule will be an improvement over a biweekly schedule. This undermines the claim that there will be the same amount of recyclables put out overall.
 LSAT1802017
  • Posts: 7
  • Joined: May 01, 2017
|
#43228
So is this attacking the premise of the argument?
What should be our prephrase? D seemed unappealing because I accepted the author's premise as the truth and anticipated that the right answer will say something that the program WILL BE more cost-effective despite people putting out the same volume of recyclables, because I organized the author's argument as:

Author: The same volume of recyclables --> NOT more cost-effective
Weaken: The same volume of recyclables ---> more cost-effective (negate the necessary part of the author's argument)

What is the most effective way to solve this Q?

Thanks!
 Claire Horan
PowerScore Staff
  • PowerScore Staff
  • Posts: 408
  • Joined: Apr 18, 2016
|
#43650
Yes, D attacks the premise that people will put out the same number of recyclables regardless of whether there are weekly or biweekly pickups.

Remember that for Weaken questions, the flow of information goes from the answer choices to the argument. In other words, accept each answer choice, in turn, as true and consider whether that weakens the argument in the stimulus. Consider reviewing this question type in the Powerscore coursebooks.

Tips for approaching this question:
I wouldn't suggest trying to diagram this question, and prephrasing isn't available here beyond something general like "something that would make the biweekly pickup more cost-effective than weekly pickup. On weaken and strengthen questions (and the other types in those two families), you won't always be able to guess the answer choices. Just consider each one, asking whether, if true, it attacks the conclusion or any premises of the argument. Once you find what you think is the right answer (because you can explain to yourself how it weakens the argument), go through the rest of the answer choices to make sure you can eliminate them.
 manchas
  • Posts: 9
  • Joined: Oct 20, 2015
|
#49422
From the previous post:
" On weaken and strengthen questions (and the other types in those two families), you won't always be able to guess the answer choices. Just consider each one, asking whether, if true, it attacks the conclusion or any premises of the argument"

This really flummoxed me. I remember hearing about subtle changes LSAC had seemed to lean towards in recent tests (from one of the recent Crystal Ball seminars that I believe Jon D conducted) but this idea of attacking a premise....Is that really a permissible thing now??? I thought it was practically holy grail that premises in the stimulus were taken to be as true. It's probably the main reason why I quickly dismissed (D), although I see why it more precisely attacks the argument in this stimulus in comparison to say, for example (C).

Thank you in advance....
User avatar
 Jonathan Evans
PowerScore Staff
  • PowerScore Staff
  • Posts: 726
  • Joined: Jun 09, 2016
|
#49609
Hi, Manchas,

This is not so much a change in the LSAT per se. It has always been possible to attack the integrity of the data offered in support of a conclusion. You are likely familiar with this happening all the time with surveys and samples used as premises. "Yes, it's true you have a survey, but it sucks."

In arguments with subordinate conclusions which are in turn used as premises for a main conclusion, it is possible to attack a fallacy that leads to the subordinate conclusion, thus calling into question the entire argument.

Overall, you are correct that we cannot flat out say "your premise did not happen," but we can call into question an underlying assumption behind the premise, thus attacking its integrity. This is what happens here. If we know the schedule is far easier to adhere to, then we have some reason to believe people may put out a greater volume of recyclables overall.

In addition, the validity of this approach to weakening the problem is heightened by the fact that we have two competing presuppositions in this argument. On the one hand, the city claims there will be more recyclables. On the other hand, the editor claims there will not be more recyclables. Who is right? We need to have some evidence to determine whose speculation is correct.

Thus, we essentially have two competing claims and no reason in particular to trust the editor's speculation more than that of the city. With the additional evidence in answer choice (D), we have good reason to go with the city over the editor, which is what we want to have happen here.

I hope this helps!
 manchas
  • Posts: 9
  • Joined: Oct 20, 2015
|
#49620
Thanks Jonathan, that helped a lot. One quick followup:

Just so I'm clear, then, is this the correct structure here;

Subordinate conclusion: people will just put out the same amount of trash, only spreading it out ..
Conclusion: So the city's claim that it will be more cost-effective is absurd.

And the correct answer challenges/calls into question that subordinate conclusion? Is that correct ?
 James Finch
PowerScore Staff
  • PowerScore Staff
  • Posts: 943
  • Joined: Sep 06, 2017
|
#58757
Hi Manchas,

I don't see any subordinate conclusion here, only a premise that flows into a conclusion that serves as a rebuttal to the city's argument. Otherwise, you have it correct: the correct answer choice is attacking the idea that the volume of recycling being collected will be the same, because an easier-to-utilize recycling program would likely mean more participation in it, leading to a greater volume of recycled waste. It's not super strong, but it does serve the purpose of weakening the editor's conclusion.

Hope this helps!
 ashnicng
  • Posts: 14
  • Joined: Jul 05, 2019
|
#67914
Could you clarify how D weakens the argument? Is D implying that if a weekly schedule is easier to adhere to, then people will recycle more often (thereby weakening the editor's argument that the volume of recyclables overall will not change?) Thanks so much!
 Jeremy Press
PowerScore Staff
  • PowerScore Staff
  • Posts: 1000
  • Joined: Jun 12, 2017
|
#67963
Hi ash,

You've got it exactly right with answer choice D! Notice also the intensifier "substantially," which lends more credence to the idea that the volume of recyclables put out will increase under the new program. If people are substantially more likely to stay on schedule ("follow and adhere to" the schedule), that just means they're substantially more likely to put their recyclables out, thus collected volume will likely increase.

I hope this helps!

Jeremy

Get the most out of your LSAT Prep Plus subscription.

Analyze and track your performance with our Testing and Analytics Package.